7
$\begingroup$

Let $A_1\leftarrow A_2\leftarrow A_3\leftarrow\dotsb$ be a projective system of abelian groups with the projection maps $p_{ij}\colon A_j\to A_i$, $j\ge i$. The derived functor of projective limit $\varprojlim_n^1 A_n$ is constructed as the cokernel of the map $$ \mathrm{id}-\mathit{shift}\colon\prod\nolimits_{n=1}^\infty A_n\longrightarrow\prod\nolimits_{n=1}^\infty A_n, $$ where the map $\mathit{shift}\colon\prod_{n=1}^\infty A_n\to\prod_{n=1}^\infty A_n$ takes a sequence $(a_n\in A_n)_{n=1}^\infty$ to the sequence $(p_{n,n+1}(a_{n+1})\in A_n)_{n=1}^\infty$. The kernel of the map $\mathrm{id}-\mathit{shift}$ is the projective limit $\varprojlim_n A_n$.

The following condition, called the Mittag-Leffler condition, is sufficient for the vanishing of $\varprojlim_n^1 A_n$. Suppose that for every fixed $m\ge1$ the nonincreasing sequence of subgroups $p_{m,n}(A_n)\subset A_m$ stabilizes for $n$ large enough. Then $\varprojlim_n^1 A_n=0$.

The Mittag-Leffler condition is not necessary for the vanishing of $\varprojlim_n^1 A_n$. The simplest counterexample would be $A_n=x^nk[[x]]\subset k[[x]]$, where $k[[x]]$ denotes the right of formal Taylor power series in one variable $x$ over a field $k$ and the maps $p_{i,j}\colon x^jk[[x]]\hookrightarrow x^ik[[x]]$ are the identity inclusions. Then $\varprojlim_n^1 A_n=\varprojlim_n A_n=0$, but the sequence of images of the maps $p_{m,n}$ never stabilizes as a sequence of subgroups in $A_m$.

Notice that, replacing the power series with the polynomials in the above example, one obtains the projective system of groups/vector spaces $C_n=x^nk[x]$ and identity inclusions between them, whose derived projective limit does not vanish. In fact, one has $\varprojlim_n^1 C_n=k[[x]]/k[x]$.

I believe I can prove the following result, which is of relevance in connection with the weak proregularity condition in the MGM duality theory. Let $(A_n)$ be a projective system of abelian groups. Denote by $(B_n)=\bigoplus_\omega (A_n)$ the direct sum of a countably infinite family of copies of the projective system $(A_n)$. Then the following three conditions are equivalent:

  1. the projective system $(A_n)$ satisfies the Mittag-Leffler condition;
  2. the projective system $(B_n)$ satisfies the Mittag-Leffler condition;
  3. $\varprojlim_n^1 B_n=0$.

My question is: this sounds like a too elementary result in a too popular area of algebra to remain unknown till the present time. Is there a reference where a proof of the equivalence of these conditions can be found?

$\endgroup$

1 Answer 1

8
$\begingroup$

This is due to Emmanouil as far as I know. See this.

$\endgroup$
0

Your Answer

By clicking “Post Your Answer”, you agree to our terms of service and acknowledge you have read our privacy policy.

Not the answer you're looking for? Browse other questions tagged or ask your own question.